Frage:
Verstrickung, real oder nur Mathe?
letmethink
2015-03-22 02:50:37 UTC
view on stackexchange narkive permalink

Bei Quantenverschränkung reagiert etwas auf ein Teilchen, das andere reagiert ebenfalls, genau umgekehrt (mehr oder weniger). Nach allem, was ich gelesen habe, wird alles, was auf eines der Teilchen wirkt, die Verstrickung aufheben, oder? Woher wissen wir also, dass sie jemals miteinander verbunden waren? Oder sind es nur Messungen, die es zusammenbrechen lassen?

Der Grund, den ich frage, ist, dass angesichts des öffentlichen Eindrucks des Themas zwei Dinge vorgeschlagen werden. Dass die Informationen, die die Reaktion verursachen, superluminal sind oder dass die Partikel denselben Raum einnehmen, da sie sich in unterschiedlichen Zuständen befinden. Wenn die zweite wahr wäre, müsste die Entfernung ein menschliches Konstrukt sein und irgendwie müssen sie sich immer noch am selben Ort befinden, unabhängig von der virtuellen Entfernung zwischen ihnen.

Zur Verdeutlichung meine ich keine Informationen in Bezug auf Kommunikation wie QC. Nur Partikelinformationen.

Wissen Sie viel über Quantenmechanik oder lineare Algebra?
Wenn Sie die Antwort eines Laien ohne Bezug auf die Mathematik der Quantenmechanik wünschen, schauen Sie sich vielleicht Brian Greenes Buch * Das Gewebe des Kosmos: Raum, Zeit und die Textur der Realität * an.Dies gibt eine ausgezeichnete und detaillierte Darstellung der Verschränkung ohne Rückgriff auf die Mathematik.
* "Bei der Quantenverschränkung, wenn etwas auf ein Teilchen einwirkt, reagiert auch das andere, genau umgekehrt (mehr oder weniger)." * Ich kann mir keine Möglichkeit vorstellen, diesen Satz so zu interpretieren, dass er korrekt ist.* "Nach dem, was ich gelesen habe, wird alles, was auf eines der Teilchen wirkt, die Verstrickung aufheben, oder?" * Ich kann mir eine Möglichkeit vorstellen, diesen Satz zu interpretieren, die ihn korrekt machen würde, aber ich bin mir nicht sicher, ob es das ist, was Sie sindim Gedächtnis haben.Bitte angeben.
* "... angesichts des öffentlichen Eindrucks des Themas ..." * Zitierweise erforderlich.Wie ist der öffentliche Eindruck?Wie formulieren Sie die Meinung zum "öffentlichen Eindruck"?
In diesem Beitrag werden im ersten Absatz mindestens * drei * verschiedene Fragen gestellt, von denen keine mit der im Titel gestellten Frage übereinstimmt. Dies ist selbst eine leere Frage, da in keiner Weise zwischen "echt" und "nur mathematisch" unterschieden wirddas kann ich mir vorstellen.In diesem Fall stimme ich dafür, als * unklar, was Sie fragen * zu schließen.Ich hoffe, der Beitrag wird bearbeitet und verbessert.
http://physics.stackexchange.com/q/281262/
Ich stimme dafür, diese Frage als nicht zum Thema gehörend zu schließen, weil sie unsinnig ist.
Mögliches Duplikat von [Quantenverschränkung und gruselige Aktion in der Ferne] (https://physics.stackexchange.com/q/126622/)
Sechs antworten:
ajctt
2015-03-22 03:47:35 UTC
view on stackexchange narkive permalink

Verschränkung ist eine Immobilie, die durch die Verletzung der Bell-Ungleichungen nachgewiesen werden kann. Dies geschieht üblicherweise so, dass ein Teilchenpaar mit verschränkten Spinzuständen in einer Konfiguration namens Glockenzustände erzeugt wird. Wenn die Verschränkung real ist, gibt mir die Messung des Zustands eines Teilchens eine genaue Kenntnis des Zustands des anderen Teilchens. Wenn es keine Verschränkung gibt, sollte die Messung eines Partikels nicht so stark mit der Messung des anderen Partikels korrelieren.

Experimentell wird festgestellt, dass die Bell-Ungleichungen verletzt werden und somit die Verschränkung real ist. Die populärste Ansicht ist, dass dies bedeutet, dass die Quantenmechanik eine genaue Darstellung der Realität ist und keine statistische Darstellung einer zugrunde liegenden deterministischen Mechanik. In dieser Ansicht ist keine überluminale Übertragung von Informationen möglich, da Sie nicht steuern können, welchen Zustand Sie erhalten, und daher nicht steuern können, was die andere Person sieht.

Eine alternative Ansicht ist, dass die Quantenmechanik eine ist Beschreibung einer nicht lokalen versteckten Variablen. Ich bin über diesen speziellen Standpunkt nicht besonders gut gelesen, aber wenn Sie an den Implikationen interessiert sind, würde ich vorschlagen, die De Broglie-Bohm-Theorie zu untersuchen.

Eine "lückenlose" freie Verletzung der Ungleichung einer Glocke bleibt eine entmutigende Aufgabe, wie hier diskutiert: http://en.wikipedia.org/wiki/Loopholes_in_Bell_test_experiments
+1 für die Antwort, ich glaube FERMLY nicht, dass QM nicht lokal ist.Sie können keine Informationen nicht lokal übertragen, und nicht lokale Theorien über versteckte Variablen sind nur monströs imo.
Wenn Sie jemals das Originalpapier von Bell gelesen haben, in dem die Ungleichungen abgeleitet wurden, wären Sie sich nicht sicher, ob eine Verletzung die Unmöglichkeit einer klassischen lokalen Theorie der versteckten Variablen zeigt.(@letmethink, @ajctt).Kurzum: Es gibt eine Prämisse, die für klassische statistische Systeme einfach nicht gilt.Daher werden die Ungleichungen auch von klassischen Systemen verletzt.Daher zeigt eine experimentelle Verletzung nichts.Ich werde in den nächsten Tagen eine Antwort darauf veröffentlichen, aber dies wird einige Zeit dauern.Der erste Teil ist unten.
Es besteht auch die Möglichkeit nicht lokaler versteckter Variablen.Mit anderen Worten, denken Sie an Parasiten, die den Pipi-Strom eines Tieres hinaufschwimmen und die Harnröhre infizieren.Was ist, wenn die Beobachtung von Alice die gesamte Wellenfunktion über ihren gesamten Bereich stört?Wir argumentieren, dass dies der Fall ist.Das muss aber nicht heißen, dass es sich nicht um einen Effekt handelt, der mit klassischen Techniken wie der Pilotwellen-Hydrodynamik physikalisch modelliert werden kann
Die Leute sprühen gerne Vitriol auf nicht lokale versteckte Variablen, aber ich glaube nicht, dass sie wirklich verstehen, was das bedeutet.Wenn man beide davon ausgeht, dass die Quantentheorie korrekt ist und jede zugrunde liegende Nichtlokalität verbietet, dann ist die einzige Option der Solipsismus: In der Tat kann man nicht einmal behaupten, dass die Wellenfunktion real ist, weil dies auch nicht lokal wäre.Man kann nicht einmal behaupten, dass Experimente selbst real sind, da sie auch nur mit Wellenfunktionen beschrieben werden können.Daher ist das einzige, was "real" sein kann, dass * Sie * es beobachten.(Und wenn man dazu getrieben wird, macht es überhaupt Sinn, 'Lokalität' zu fordern?)
pyramids
2015-03-22 03:49:17 UTC
view on stackexchange narkive permalink

Auf dieses Problem wurde historisch in dem heute allgemein als EPR-Papier abgekürzten Problem hingewiesen, für das ich Sie einfach auf eine Antwort auf eine sehr ähnliche Frage verweise. Dieser scheinbar paradoxe Effekt wurde experimentell beobachtet.

Einige Leute bestehen darauf, dass die Frage, ob er "real" ist, noch ungelöst ist. Die Hauptschwierigkeit besteht jedoch einfach darin, dass die Frage, was "real" ist, eine Frage der Philosophie und nicht der (Natur-) Wissenschaft ist. Stiffler für Details unter Physikern nennen daher die verschiedenen Denkweisen, die streng genommen nicht im Widerspruch zu unserer mathematischen Beschreibung der Quantenmechanik Interpretationen stehen. Bei der Wahl einer Interpretation kann man nicht gleichzeitig Lokalität (die Idee, dass die Zustände der Teilchen irgendwie in ihnen liegen, was superluminale Effekte impliziert, oder eine "gruselige Fernwirkung", wie Einstein sie angeblich nannte) und Realismus (Ihre Frage, wenn es so ist) haben ist echt"). CAVEAT: Die Wikipedia-Autoren für den Interpretationslink scheinen nicht zuzustimmen, oder zumindest die Übersichtstabelle schlägt dies vor. Ich stütze meine Aussage auf die Argumentation von Nielsen und Chuang in ihrem Lehrbuch Quantenberechnung und Quanteninformation.

Wenn Sie nur eine Interpretation wünschen, um alles zu verstehen, würde ich empfehlen Ein informativer Ansatz: Stellen Sie sich Zustände nicht als (lokale) Eigenschaften von Partikeln vor. Stattdessen beschreiben sie Ihre Informationen über das System: Wenn Sie eine geeignete Überlagerung zwischen zwei Partikeln vorbereitet haben, wissen Sie, dass sich das Partikel in jedem Zustand befindet, in dem sich das andere befindet. Das sind globale Informationen, die bei der Interaktion der Partikel entstehen und keine überluminalen Effekte erfordern. Anstatt dass ein Teilchen plötzlich seinen Zustand ändert, liegt der Effekt im (globalen) Wissen über das System.

Der andere Aspekt Ihrer Fragen betrifft das Problem des Zusammenbruchs der Wellenfunktion. Auch dies ist eine Interpretation, und wie Sie anscheinend bereits erkennen, keine sehr konsistente (oder zumindest nicht umfassende). Das Paradoxon verschwindet, wenn Sie Ihre Interaktion nicht als eine Art Messung (was auch immer das ist) modellieren, sondern als quantenmechanische Interaktion mit einem anderen Teilchen. Dies führt zu einer zusätzlichen Verschränkung (mit diesem neuen Partikel oder auch mit der Umgebung). Dies bedeutet, dass die Wellenfunktionen, die nur Ihre Partikel ohne diese beschreiben, nicht wie zuvor stören, wenn Sie die Umgebung von Ihrer Analyse ausschließen. Wenn diese Störung vollständig zerstört wird, haben Sie im System ohne die Umgebung die gleichen beobachtbaren Auswirkungen, als ob Sie Wellenfunktionen kollabieren würden.

image
2015-03-22 06:18:21 UTC
view on stackexchange narkive permalink

Bei der Verschränkung geht es nicht um Interaktion oder Informationsübertragung zwischen verschränkten Partikeln.

Betrachten Sie die Spinverschränkung zweier Spin - $ \ frac {1} {2} $ -Partikel: Lassen Sie sie sich im Singulettzustand befinden relativ zu einer beliebigen Achse (z. B. Z-Achse):

$$ | \ Psi \ rangle = \ frac {1} {\ sqrt {2}} (\ | \ uparrow_z, \ downarrow_z \ rangle - | \ downarrow_z, \ uparrow_z \ rangle \) $$

Die Fähigkeit $ P $, beide Partikel im Zustand $ | i zu messen, j \ rangle $ mit $ i, j \ in \ {\ uparrow, \ downarrow \} $ wobei die Achse beider Messungen den Winkel $ \ theta $ einschließt, ist gegeben durch: $$ P_ {i, j} = \ | \ langle i, j | \ Psi \ rangle \ | ^ 2 = \ frac {1} {4} (1 - i \ cdot j \ cdot \ cos \ theta) $$ Wenn wir $ i nehmen, ist j $ 1 und -1 für $ \ uparrow $ bzw. $ \ downarrow $.

Die reduzierte Propability $ p_i $, nur ein Teilchen zu messen (z. B. wenn wir uns nicht um das andere kümmern), ist gegeben durch : $$ p_i = \ sum_ {j \ in \ {1, -1 \}} P_ {i, j} = \ frac {1} {2} $$

Die Bedingung Die Wahrscheinlichkeit, das andere Teilchen zu messen ( nachdem wir bereits über die Messung des ersten Teilchens Bescheid wissen), ist gegeben durch: $$ \ tilde {p} _ {j | i} = \ frac { P_ {i, j}} {p_i} = \ frac {1} {2} (1 - i \ cdot j \ cdot \ cos \ theta) $$

Dies beinhaltet den Winkel $ \ theta $ und normalerweise beginnt man hier damit, über Nichtlokalität und augenblickliche Aktionen zu streiten, die das Ergebnis des Experiments verändern, wenn wir den Winkel $ \ theta $ am ersten Messgerät ändern.

Dies ist jedoch nicht wahr. Wenn es sich um bedingte Wahrscheinlichkeiten handelt, haben wir bereits eine Messung durchgeführt und die Messachse der ersten Messung festgelegt. Das anschließende Ändern dieser Achse wirkt sich nicht auf die Propabilität aus, da der Winkel $ \ theta $ relativ zur gemessenen Achse ist. Durch Ändern der Achse der zweiten Messung wird nur die Wahrscheinlichkeit geändert, die das Ergebnis der späteren Messung für den ersten Beobachter vorhersagt, da er über dieses zusätzliche Wissen verfügt.

Die Propability für den zweiten Beobachter bleibt gleich, da dies die reduzierte Propability ist (er weiß nichts über die erste Messung): $$ p_j = \ sum_ {i \ in \ { 1, -1 \}} P_ {i, j} = \ frac {1} {2} $$

Kurz gesagt: Ohne das zusätzliche Wissen über die erste Messung ist die Verschränkung für die zweite nicht wichtig Beobachter. Um dieses zusätzliche Wissen zu erlangen, muss ein zusätzlicher Informationstransfer zum zweiten Beobachter erfolgen, der durch Relativität-Kausalität ($ v \ le c $ usw.) eingeschränkt wird. Verschränkung bricht also weder die Kausalität noch kann sie Informationen übertragen.

$$$$

Manchmal kommt man zu dem Argument, dass die Verletzung von Bell-Ungleichungen zeigt, dass Verschränkung immer noch etwas mehr ist als es die klassische Wahrnehmung erlauben würde. Schauen wir uns also einen bestimmten Erwartungswert an. Die Achse für die Spinmessung ist mit normalisierten Vektoren $ \ vec {a} $ und $ \ vec {b} $ zu kennzeichnen, so dass $ \ vec {a} \ cdot \ vec {b} = \ cos \ theta $. Betrachten Sie \ begin {Gleichung} \ langle \ Psi | \ vec {a} \ cdot \ vec {S_1} \ \ \ vec {b} \ cdot \ vec {S} _2 | \ Psi \ rangle = - \ frac {\ hbar ^ 2} {4} \ vec {a} \ cdot \ vec {b} = - \ frac {\ hbar ^ 2} {4} \ cos \ theta \ tag {1 } \ end {Gleichung}, dies ist der Erwartungswert des Produkts beider Messergebnisse. Hier haben wir $ \ vec {S} = \ frac {\ hbar} {2} (\ sigma_x, \ sigma_y, \ sigma_z) ^ T $ mit $ \ sigma_x, \ sigma_y, \ sigma_z $ die Pauli-Matrizen. Wir folgen jetzt der Argumentation von John Bell in seinem Originalwerk, da andere, ähnliche Ungleichungen auf demselben Problem beruhen.

Das Argument lautet wie folgt: Nehmen Sie ein klassisches statistisches System mit nicht versteckten und versteckten Variablen an, die alle mit $ \ vec {\ lambda} = (\ lambda_1, \ dots, \ lambda_n) $ für einige $ n \ in gekennzeichnet sind \ mathbb N $. Darüber hinaus gibt es zwei Funktionen $ A (\ vec {a}, \ vec {\ lambda}) $ und $ B (\ vec {b}, \ vec {\ lambda}) $, die die Ergebnisse der Spinmessung an Partikel 1 liefern bzw. 2. Sie können nur $ \ pm \ frac {\ hbar} {2} $ ergeben, da dies das einzige Ergebnis des Experiments ist. Diese Funktionen hängen nur von einer Messachse ab, da zwischen den Messgeräten 1 und 2 keine Aktion stattfinden darf (dies ist die angenommene Lokalität ).

$$$$

Da das System auf statistischer Basis untersucht wird, existiert eine Propability-Dichte $ \ varrho (\ vec {\ lambda}) $, die eine Funktion der Systemparameter $ \ vec {ist. \ lambda} $ und ermöglicht die Berechnung des Erwartungswerts $$ E (\ vec {a}, \ vec {b}) = \ int \ varrho (\ vec {\ lambda}) \ cdot A (\ vec {a}, \ vec {\ lambda}) B (\ vec {b}, \ vec {\ lambda}) \ d ^ n \ lambda $$, was dem von oben (1) entsprechen sollte, wenn dies der Fall ist ist auf klassischer, lokaler Basis zu interpretieren (Hinweis: Man kann diskrete statistische Variablen durch Begriffe wie $ \ sum_j \ alpha_j \ cdot \ delta (c_j- \ lambda_m) ​​$ einbeziehen). Die böswillige Annahme hier ist, dass $ \ varrho $ keine Funktion der Achsenvektoren $ \ vec {a} $ und $ \ vec {b} $ ist. Dies ist jedoch für klassische Systeme mit Korrelation ganz natürlich. Der Punkt ist: Zulassen von $ \ varrho (\ vec {\ lambda}, \ vec {a}, \ vec {b}) $ oder sogar nur $ \ varrho (\ vec {\ lambda}, \ vec {a} \ cdot \ vec {b}) $, die Bellschen Ungleichungen können nicht abgeleitet werden! Solche Propabilitätsdichten können eine Verletzung der Ungleichung verursachen. Um das zu verstehen, werde ich sie jetzt ableiten und darauf hinweisen, welcher Schritt mit der modifizierten Dichte nicht möglich ist:

$$$$

Angenommen, $$ E (\ vec {a}) , \ vec {b}) = - \ frac {\ hbar ^ 2} {4} \ vec {a} \ cdot \ vec {b} \ tag2 $$, so dass die quantenmechanische Beschreibung mit der klassischen übereinstimmt. Für $ \ vec {a} = \ vec {b} $: \ begin {Gleichung} \ begin {align} - \ frac {\ hbar ^ 2} {4} & = \ int \ underbrace {\ varrho (\ vec { \ lambda})} _ {\ ge 0} \ cdot \ underbrace {A (\ vec {a}, \ vec {\ lambda}) B (\ vec {a}, \ vec {\ lambda})} _ {\ ge - \ frac {\ hbar ^ 2} {4}} \, d ^ n \ lambda \\ & \ Leftrightarrow \\ 0 & = \ int \ underbrace {\ varrho (\ vec {\ lambda})} _ {\ ge 0} \ cdot \ left (\ underbrace {A (\ vec {a}, \ vec {\ lambda}) B (\ vec {a}, \ vec {\ lambda}) + \ frac {\ hbar ^ 2} {4}} _ {\ ge 0} \ right) \, d ^ n \ lambda \ end {align} \ end {Gleichung}, da $ \ varrho $ eine normalisierte Propabilitätsdichte ist. Daraus folgt, dass \ begin {Gleichung} \ begin {ausgerichtet} A (\ vec {a}, \ vec {\ lambda}) B (\ vec {a}, \ vec {\ lambda}) = - \ frac {\ hbar ^ 2} {4} \ end {align} \ end {Gleichung} ist eine gültige Gleichung unter dem Integral mit $ \ varrho $. Dies kann nur gelten, wenn \ begin {Gleichung} \ begin {ausgerichtet} B (\ vec {a}, \ vec {\ lambda}) = - A (\ vec {a}, \ vec {\ lambda}) \ end { ausgerichtet} \ tag3 \ end {Gleichung}. Beachten Sie, dass dies für jeden Vektor $ \ vec {a} $ gilt. Nehmen Sie nun einen anderen normalisierten Vektor $ \ vec {c} $ und führen Sie die folgenden Berechnungen durch: \ begin {align} \ frac {\ hbar ^ 2} {4} | (- \ vec {a} \ cdot \ vec {b}) - (- \ vec {a} \ cdot \ vec {c}) | & = | E (\ vec {a}, \ vec {b}) - E (\ vec {a}, \ vec {c}) | \\ & = \ left | - \ int \ varrho (\ vec {\ lambda}) \ cdot (A (\ vec {a}, \ vec {\ lambda}) A (\ vec {b}, \ vec {\ lambda}) - A (\ vec {a}, \ vec {\ lambda}) A (\ vec {c}, \ vec {\ lambda})) \, d ^ n \ lambda \ right | \\ & = \ left | \ int \ varrho (\ vec {\ lambda}) \ cdot A (\ vec {a}, \ vec {\ lambda}) A (\ vec {b}, \ vec {\ lambda}) \ cdot (1 - \ frac {4} {\ hbar ^ 2} A (\ vec {b}, \ vec {\ lambda}) A (\ vec {c}, \ vec {\ lambda})) \, d ^ n \ lambda \ right | \\ & \ le \ int | \ varrho (\ vec {\ lambda}) | \ cdot | A (\ vec {a}, \ vec {\ lambda}) A (\ vec {b}, \ vec {\ lambda}) | \ cdot | 1 - \ frac {4} {\ hbar ^ 2} A (\ vec {b}, \ vec {\ lambda}) A (\ vec {c}, \ vec {\ lambda}) | \, d ^ n \ lambda \\ & = \ int \ varrho (\ vec {\ lambda}) \ cdot (\ frac {\ hbar ^ 2} {4} - A (\ vec {b}, \ vec {\ lambda}) A (\ vec {c }, \ vec {\ lambda})) \, d ^ n \ lambda \\ & = \ frac {\ hbar ^ 2} {4} + E (\ vec {b}, \ vec {c}) = \ frac {\ hbar ^ 2} {4} - \ frac {\ hbar ^ 2} {4} \ vec {b} \ cdot \ vec {c} \ tag4 \ end {align}

Im ersten Gleichheit haben wir (2) verwendet. Im zweiten haben wir (3) verwendet. Im dritten haben wir $ A (\ vec {b}, \ vec {\ lambda}) ^ 2 = \ frac {\ hbar ^ 2} {4} $ verwendet. Der vierte Schritt ist die Dreiecksungleichung für Integrale. Im fünften Schritt haben wir $ A (\ vec {a}, \ vec {\ lambda}) A (\ vec {b}, \ vec {\ lambda}) = \ pm \ frac {\ hbar ^ 2} {4 verwendet } $ und $ \ varrho (\ vec {\ lambda}) \ ge 0 $. Im letzten Schritt haben wir (2) und die Tatsache verwendet, dass $ \ varrho $ normalisiert ist.

Wir haben also endlich die Bellsche Ungleichung \ begin {Gleichung} \ begin {ausgerichtet} | \ vec {a} \ cdot \ vec {b} - \ vec {a} \ cdot \ vec {c} | + \ vec {b} \ cdot \ vec {c} \ le 1 \ ,, \ end {align} \ tag5 \ end {Gleichung}, die für eine Auswahl von $ \ vec {a}, \ vec {verletzt werden kann b}, \ vec {c} $. Dies zeigt normalerweise, dass unsere erste Annahme (2) falsch ist. Daher sollte kein klassisches lokales System in der Lage sein, den Erwartungswert (1) zu beschreiben.

$$$$

Mit der modifizierten Wahrscheinlichkeitsdichte der Die Schritte in (4) sehen folgendermaßen aus: \ begin {align} \ frac {\ hbar ^ 2} {4} | (- \ vec {a} \ cdot \ vec {b}) - ( - \ vec {a} \ cdot \ vec {c}) | & = | E (\ vec {a}, \ vec {b}) - E (\ vec {a}, \ vec {c}) | \ notag \\ & = \ left | - \ int \ varrho (\ vec {\ lambda}, \ vec {a}, \ vec {b}) \ cdot A (\ vec {a}, \ vec {\ lambda}) A (\ vec {b}, \ vec {\ lambda}) - \ varrho (\ vec {\ lambda}, \ vec {a}, \ vec {c}) \ cdot A (\ vec {a}, \ vec {\ lambda}) A (\ vec {c}, \ vec {\ lambda}) \, d ^ n \ lambda \ right | \ notag \\ & = \ left | \ int A (\ vec {a}, \ vec {\ lambda}) A (\ vec {b}, \ vec {\ lambda}) (\ varrho (\ vec {\ lambda}, \ vec {a}, \ vec {b}) - \ varrho (\ vec {\ lambda}, \ vec {a}, \ vec {c}) \ frac {4} {\ hbar ^ 2} A (\ vec {b}, \ vec { \ lambda}) A (\ vec {c}, \ vec {\ lambda})) \, d ^ n \ lambda \ right | \keine Markierung \\ & \ le \ int \ frac {\ hbar ^ 2} {4} \ cdot \ left | \ varrho (\ vec {\ lambda}, \ vec {a}, \ vec {b}) - \ varrho (\ vec {\ lambda}, \ vec {a}, \ vec {c}) \ frac {4} {\ hbar ^ 2} A (\ vec {b}, \ vec {\ lambda}) A (\ vec {c}, \ vec {\ lambda}) \ right | \, d ^ n \ lambda \ end {align} Beachten Sie, dass man nicht von hier aus fortfahren kann, da im Allgemeinen $ \ varrho (\ vec {\ lambda}, \ vec {a}, \ vec {b }) \ ne \ varrho (\ vec {\ lambda}, \ vec {a}, \ vec {c}) $. Auch die zweite Gleichheit sollte hier sowieso nicht funktionieren, da (3) nur dann gültig ist, wenn sie mit $ \ varrho (\ vec {\ lambda}, \ vec {a}, \ vec {a}) multipliziert wird. $. Zum Beispiel, wenn $ \ varrho (\ vec {\ lambda}, \ vec {a}, \ vec {a}) = 0 $ Gleichung (3) im Allgemeinen verletzt werden kann. Trotzdem konnte man nur versuchen, eine andere Dreiecksgleichung für den Begriff $ | \ dots | $ zu verwenden, so dass wir schließlich die Ungleichung \ begin {Gleichung} \ begin {align} | \ vec {a} \ cdot \ vec {b} haben - \ vec {a} \ cdot \ vec {c} | \ le 2 \ ,, \ end {ausgerichtet} \ end {Gleichung}, die von keiner Auswahl von $ \ vec {a}, \ vec {b}, \ vec {c} $ verletzt werden darf.

$$$$

Zusammenfassend: Wenn man Propabilitätsdichten $ \ varrho (\ vec {\ lambda}, \ vec {a}, \ vec {b}) $ zulässt, die davon abhängen Bei einigen Messparametern ist die Ableitung einer Ungleichung, die durch quantenmechanische Erwartungswerte verletzt wird, nicht in üblicher Weise möglich. Oben habe ich bereits argumentiert, dass die Abhängigkeit von $ \ vec {a}, \ vec {b} $ im Allgemeinen kein Grund für nicht lokales Verhalten ist, solange die reduzierte Propabilität eines Subsystems nur besteht hing von seinen eigenen Parametern ab. Dieses Problem ist auf Ungleichungen zurückzuführen, die auf denselben Argumenten wie die Bellsche Ungleichung beruhen: siehe zum Beispiel die CHSH-Ungleichung auf Seite 527, Gleichung 2, die häufig in Experimenten verwendet wird!

$$$$

Wenn wir also einige Funktionen $ A $ und $ B $ finden würden, die unsere Lokalitätsbedingungen von oben erfüllen, gibt es keinen Grund, an den Erwartungswert (1) zu denken. als nicht lokal. Nimm \ begin {align} p_ {i, j} (\ vec {a}, \ vec {b}) & = \ frac {1} {4} (1 - ij \ \ vec {a} \ cdot \ vec {b}) \\ A. (i, \ vec {a}) & = \ frac {\ hbar} {2} \ i \\ B (j, \ vec {b}) & = \ frac {\ hbar} {2} \ j \ end { align} Dann haben wir $$ E (\ vec {a}, \ vec {b}) = \ sum_ {i, j \ in \ {1, -1 \}} p_ {i, j} (\ vec {a }, \ vec {b}) \ cdot A (i, \ vec {a}) B (j, \ vec {b}) = - \ frac {\ hbar ^ 2} {4} \ \ vec {a} \ cdot \ vec {b} = - \ frac {\ hbar ^ 2} {4} \ \ cos \ theta $$, was auf reiner, lokaler und klassischer Basis (1) entspricht.

Hinweis: Diese Antwort wird in den nächsten Tagen auf ein Argument gegen Bell-Ungleichungen ausgeweitet.
Aus Ihrem obigen Kommentar zu Ajctts Antwort geht hervor, dass einer Ihrer Hauptpunkte darin besteht, dass die Ungleichungen von Bell selbst durch klassische (lokale, reale) Systeme verletzt werden könnten.Während Sie Ihre Antwort erweitern, wäre es großartig, wenn Sie diesen Punkt weiter veranschaulichen könnten, idealerweise anhand eines konkreten Beispiels.Das wäre sehr interessant.
Nur zu Ihrer Information - Ich stelle mir vor, dass diese Antwort nützlich ist, aber sie ist zu "dicht", um für jemanden nützlich zu sein, der nicht mit dem Fachgebiet, der Terminologie und der Symbologie vertraut ist.(zB habe ich einen Master-Abschluss in einem nicht verwandten technischen Bereich und einen "mäßig hohen" IQ, aber ich würde in diese Kategorien aufgenommen).Das ist in Ordnung genug, wenn Sie ein sehr begrenztes Publikum ansprechen möchten, aber wenn Sie möchten, dass es intelligenten kompetenten Wissenschaftlern (und Ingenieuren (einschließlich mir :-))) zugänglich ist, die nicht alle oben genannten Kästchen ankreuzen, dann braucht eszusätzliches Erklärungsmaterial. Ob dies als wertvoll erachtet wird, liegt bei Ihnen.
@RussellMcMahon: Ich verstehe, was Sie meinen, aber eine vage und unklare Sprache ohne angemessene Definitionen in diesem Bereich (und allgemein in der Physik) führte zu einer Situation, für die einige Theorien von der öffentlichen Gemeinschaft viel zu schnell aufgegeben wurden.Aus diesem Grund scheinen meine Antworten für Personen, die nicht direkt an diesem Thema beteiligt sind, etwas "außerhalb des Rahmens" zu liegen, da ich versuche, so genau wie möglich zu sein.Der einzige Weg, dies zu tun, ist durch Symbole und Terminologie.Dennoch sind Symbole niemals bedeutungslos.Wenn Sie eine bestimmte Frage dazu haben, werde ich versuchen, sie zu beantworten.
@Emil: der Hauptteil ist fertig.Das Gedankenexperiment steht noch bevor
Schöne Antwort, ich freue mich auf das Gedankenexperiment, denn momentan sieht $ \ rho (\ lambda, \ vec a, \ vec b) $ für mich * sehr * unklassisch aus - eine Wahrscheinlichkeitsverteilung, die davon abhängt, was wir wollenmessen?!
Danke für die Ergänzungen.Sehr interessant.Ich denke, Sie haben da einen Punkt.Meines (ziemlich begrenzten) Wissens ist dies effektiv die Bell-Lücke von "keine Wahlfreiheit".Das heißt, Sie können den lokalen Realismus bewahren, wenn Sie nicht die Möglichkeit absolut unabhängiger Messgeräte benötigen.Beachten Sie, dass dies auch die Möglichkeit von echten Zufallszahlengeneratoren verweigern würde.Einige Leute finden dies unerwünscht und würden den Realismus lieber aufgeben.Verstöße gegen die Bellschen Ungleichungen zeigen jedoch nur, dass wir nicht alle drei von Lokalität, Realismus und Freiheit haben können.Zwei von ihnen sind in Ordnung.
@Emil: Nein, das versuche ich nicht zu zeigen.Man braucht wirklich keine Lücken, da die Bellschen Ungleichungen auch für klassische, lokale, reale Systeme einfach nicht gelten.Es geht um die Missverständnisse in der Prämisse für $ \ varrho $
@ACuriousMind: Die Propabilitätsverteilung hängt nicht davon ab, was wir messen möchten, da $ \ vec {a} $ nicht Teil des Messergebnisses ist.Es stimmt nur damit überein, dass die Einstellung des Messgerätes die Propabilität beeinflussen kann (nicht muss!).Nehmen wir zum Beispiel ein Teilchen, das immer auf die x-Achse gerichtet ist.Wenn wir entlang der y-Achse messen, werden wir es nicht finden, also $ \ varrho (\ lambda, x-Achse) \ ne \ varrho (\ lambda, y-Achse) = 0 $
@Marcel Wenn Sie sich den letzten Absatz in Abschnitt III, "Illustration", von Bells Originalwerk (http://philoscience.unibe.ch/documents/TexteHS10/bell1964epr.pdf) ansehen, werden Sie feststellen, dass er den Fall ausgeschlossen hat, dass Sievorgeschlagen, als nicht-lokaler Fall.In seinen Worten "gibt es keine Schwierigkeit, die quantenmechanische Korrelation (3) zu reproduzieren, wenn die Ergebnisse A und B in (2) von $ \ vec {b} $ bzw. $ \ vec {a} $ als abhängen dürfensowie $ \ vec {a} $ und $ \ vec {b} $. "
@user36125: nein, ich habe die Beschränkung von $ A $ und $ B $ nicht angegangen.Ich habe die "Einschränkung" von $ \ varrho $ angegangen, von der er eigentlich nichts wusste.
@Marcel Ich kritisiere Ihre Idee nicht, erinnere Sie jedoch nur daran, dass er und viele veröffentlichte Artikel * DID * per Definition auf den Fall verweisen, auf den Sie als _non-local_ hingewiesen haben.Ich hatte genau den gleichen Gedanken, auf den Sie hingewiesen haben.Die Bell "Nichtlokalität" ist jedoch bereits seit fast einem halben Jahrhundert eine akzeptierte Definition (oder ein schlecht benanntes Konzept).Es ist zu spät, um die alte Definition zu ändern;)
@user36125: Die Einschränkung für $ \ varrho $ entspricht jedoch in keiner Weise einem Lokalitätsargument.Es ist * keine * gültige Einschränkung für ein lokales klassisches System.Ich würde sagen, es ist nur ein Fehler in der Geschichte, an dem wir festgehalten haben.
Ich stimme vollkommen zu, dass es ein Fehler in der Geschichte ist.Das Schlimmste ist, dass es viele professionelle Physiker lange Zeit verwirrt hatte.Es ist wahrscheinlich nur ein nicht intuitives Wahrscheinlichkeitsproblem, das vielleicht eines Tages geklärt wird.
@Marcel: Haben Sie diese Analyse veröffentlicht?
@Marcel: Bitte kommen Sie hierher, um es bekannt zu geben.TY
@igael: Ich würde das wirklich gerne tun.Ich habe [hier] einen Chatraum eingerichtet (http://chat.stackexchange.com/rooms/43158/entanglement).
@Marcel: Ich kann nicht chatten, sorry, ich schreibe so langsam.arbeite noch daran.Neue Synthese ohne vorgefertigte Schlussfolgerung oder Leidenschaft: Es ist genau die Antwort, die dieses Thema benötigt ...
Jimmy360
2015-03-22 03:40:12 UTC
view on stackexchange narkive permalink

Es ist real und experimentell verifiziert. Wenn Sie den Spin von verwickeltem Partikel A messen, hat Partikel B bei der Messung immer den entgegengesetzten Spin. Einige Physiker glauben, dass dies mit superluminaler Kommunikation zu tun hat, aber es gibt viele andere Theorien.

Yogi DMT
2016-10-05 01:58:24 UTC
view on stackexchange narkive permalink

Woher wissen wir also, dass sie jemals verknüpft wurden?

Weil die Ergebnisse gegen Bellsche Ungleichungen verstoßen.

Oder sind es nur Messungen, die es zusammenbrechen lassen?

Das Messen eines verschränkten Systems zerstört die Verschränkung.

Die Ergebnisse von Experimenten mit verschränkten Partikeln sind nur möglich mit:

A) FTL-Informationen (ich sage "Raum ist nicht das, was wir denken" -Theorien in derselben Kategorie, weil zwischen beiden eine große Grauzone liegt). Ich werde auch erwähnen, dass ich keine Lust auf Theorien habe, die versuchen, dies zu umgehen (dh Pilotwellentheorie). Für mich gehört die Nichtlokalität zur breiteren Kategorie der FTL.

oder B) Superdeterminismus: "Teilchen A weiß, wie wir uns entscheiden werden, Teilchen B zu messen, bevor wir es messen, und ja, ich weiß, dass es weit hergeholt ist, aber es muss als einzige andere Alternative erwähnt werden.

Verschränkung ist definitiv real und nicht nur Mathe. Das Lesen dieses Wiki-Links bietet möglicherweise weitere Informationen.

kpv
2016-09-26 10:48:20 UTC
view on stackexchange narkive permalink

OP schrieb: "Bei der Quantenverschränkung, wenn etwas auf ein Teilchen einwirkt, reagiert auch das andere, genau umgekehrt (mehr oder weniger)."

Diese Beobachtung könnte auf eine "perfekte Antikorrelation" hindeuten. d.h. wenn der Spin von zwei "verschränkten" Teilchen in demselben Winkel gemessen wird, ist garantiert, dass sie entgegengesetzt sind. Es gibt einige spezifische Zustände, in denen verschränkte Paare, die in diesem Zustand erzeugt / hergestellt werden, eine perfekte Antikorrelation aufweisen. (Wenn es keine solchen Zustände gibt, sachkundige QM-Leute, bitte kommentieren). Der Staat kann einer der Bell-Staaten sein - https://en.wikipedia.org/wiki/Bell_state#The_Bell_states

Die Antikorrelation unterscheidet sich stark von der statistischen Korrelation, und das Vermischen der beiden führt zu aller Verwirrung.

QM weist Durchschnittswerte / Wahrscheinlichkeiten zu. Es gibt nichts wie 0 oder 1 Wahrscheinlichkeit, was bedeutet, dass es kein garantiertes Ergebnis in der Wahrscheinlichkeit gibt. Das bedeutet, dass eine perfekte Antikorrelation (des Spins) eher durch ein Gesetz als durch eine Wahrscheinlichkeit erzwungen wird.

Dieses Gesetz ist die Erhaltung des Drehimpulses. Wenn die Teilchen zu diesem Zeitpunkt hergestellt werden, werden sie so vorbereitet, dass sie, um den Drehimpuls in jeder Richtung zu erhalten, codiert (versteckte Variablen) mit entgegengesetztem Spin in jede Richtung codiert werden. Wann und wo immer sie gemessen werden. Distanz ist hier also nicht einmal ein Faktor.

Daher müssen sie nicht denselben Platz belegen und müssen überhaupt nicht kommunizieren, geschweige denn mit FTL-Geschwindigkeit. Die Antikorrelation ist also eine direkte Folge der Erhaltungsgesetze. Die Antikorrelation ist eine Beziehung zwischen den Ergebnissen zweier Partikel desselben Paares

Das Problem tritt auf, wenn Menschen Antikorrelationsphänomene verwenden, um statistische Korrelationen zu erklären. Sie zählen versteckte Variablen basierend auf der Antikorrelation auf und berechnen die Bellsche Ungleichung basierend auf diesen Aufzählungen und wenden die Ungleichung auf statistische Korrelationen an. An diesem Punkt vergessen sie zu berücksichtigen, dass die statistische Korrelation zwischen den Ergebnissen von VARIOUS PAIRS b> besteht, während die Antikorrelation zwischen den Ergebnissen von SAME PAIR b> besteht. Daher ist die Anwendung der Bellschen Ungleichung auf statistische Korrelationen ein grober Missbrauch eines mathematischen Theorems.

Die Antikorrelation wird durch Erhaltungsgesetze erklärt.

Die statistische Korrelation sollte unabhängig von Antikorrelationen überprüft und erklärt werden. Die Vermischung von Antikorrelation mit statistischer Korrelation ist die Wurzel des ganzen Rätsels, das die Verschränkung umgibt.

Die statistische Korrelation ist ein Spiel mit Durchschnittswerten und muss überprüft und anhand experimenteller Faktoren erklärt werden. Faktoren, die als Schlupflöcher bezeichnet werden Diese Faktoren müssen statistische Korrelationen über die Dauer des Experiments bilden. Wenn dies der Fall ist, gibt es keine Verstrickung.

In einem ersten Schritt habe ich Daten aus einem kürzlich durchgeführten Verschränkungsexperiment analysiert. Diese Analyse gibt einen Hinweis darauf, dass statistische Korrelationen nicht unbedingt von Wahrscheinlichkeiten geleitet werden. Es kann unter http://vixra.org/abs/1609.0237 gelesen werden.

Es gibt unendlich viele Beispiele für "Verschränkung", was, wie Sie sagen, eine logische Schlussfolgerung ist, die von Gesetzen im klassischen Alltag abhängt."Wenn John und Harry Smith Brüder sind und Sie hören, dass einer von ihnen ins Ausland gegangen ist, wenn Sie John auf der Straße treffen, wissen Sie sofort, dass Harry im Ausland ist."
* "Wenn der Spin von zwei" verschränkten "Partikeln im gleichen Winkel gemessen wird, sind sie garantiert entgegengesetzt." * Das ist sicherlich falsch.
-1
@DanielSank:Voll verwickelter Spin 1/2 Zustand .... Ich weiß nicht, wie ich das beschreiben soll, aber ich hoffe, Sie wissen, worauf hier Bezug genommen wird.
Was ist mit $ | \ uparrow \ uparrow \ rangle + | \ downarrow \ downarrow \ rangle $.
@DanielSank: Um ehrlich zu sein, bin ich völlig behindert darin, die Staaten und den Formalismus zu kennen, also verstehe ich nicht, was Sie geschrieben haben.Es wird jedoch auf einen vollkommen antikorrelierten Zustand Bezug genommen, zum Beispiel auf den in http://physics.stackexchange.com/questions/31141/why-are-there-only-perfectly-anti-correlated-quantum-states-.nicht perfekt korrigiert
@kpv Wenn Ihnen der Punkt, den Daniel gerade angesprochen hat, nicht klar ist, sollten Sie ernsthaft über die Möglichkeit nachdenken, dass Sie das Thema einfach nicht genug verstehen, um diese Frage zu beantworten.(Falls Sie es verpasst haben, hat Daniel Ihnen nur ein klares Beispiel für einen maximal verschränkten Zustand gegeben, der * nicht * perfekt antikorreliert ist.) Dies ist derzeit eine entsetzlich schlechte Antwort auf die Frage und kaum mehr als eine Werbung (Spam?) für das Papier, das Sie auf dieser Seite weiter pushen.
@EmilioPisanty: Ich habe keine Probleme damit, meine Antwort abzustimmen.Lassen Sie mich einfach wissen, ob es einen Zustand / ein Szenario gibt, in dem die Partikel des verschränkten Paares perfekt antikorreliert sind.Unabhängig vom Zustand oder wie Sie es schreiben, wie hoch ist die höchste Wahrscheinlichkeit einer Antikorrelation in einem verwickelten Paar?Wenn es keine perfekte Antikorrelation in der Verschränkung gibt, werde ich diese Antwort löschen.Aber bitte lassen Sie mich den höchstmöglichen Antikorrelationsprozentsatz wissen.Wenn es ein Szenario perfekter Antikorrelation gibt, lassen Sie mich wissen, wie es geschrieben ist.
@EmilioPisanty: Oben auf Seite 8 von http://www.theory.caltech.edu/people/preskill/ph229/notes/chap4.pdf heißt es: "wobei θ der Winkel zwischen den Achsen ˆn und ˆm ist. Somit stellen wir fest, dass die Messergebnissesind immer perfekt antikorreliert, wenn wir beide Drehungen entlang derselben Achse ˆn messen ".Es scheint, als gäbe es ein Szenario perfekter Antikorrelation.Bitte lassen Sie mich wissen, wenn dies nicht korrekt ist.
@EmilioPisanty: Auch https://en.wikipedia.org/wiki/Bell%27s_theorem spricht von perfekter Antikorrelation, obwohl ich nicht sicher bin, ob es sich um ein reales Szenario oder eine Hypothese handelt, die Sie besser beurteilen können."Angenommen, die beiden Partikel sind perfekt antikorreliert - in dem Sinne, dass immer dann, wenn beide in die gleiche Richtung gemessen werden, identisch entgegengesetzte Ergebnisse erzielt werden, wenn beide in entgegengesetzten Richtungen gemessen werden, immer das gleiche Ergebnis erzielt wird. Nur so kann man sich vorstellen, wie dies funktioniertist, dass beide Teilchen ihre gemeinsame Quelle mit den Ergebnissen verlassen, die sie liefern, wenn sie in einem beliebigen Punkt gemessen werden. "
@EmilioPisanty:Seite 100vonhttps://books.google.com/books?id=x_oODQAAQBAJ&pg=PA100&lpg=PA100&dq=perfect+anti+correlation&source=bl&ots=tZahsXNtUG&sig=6ZsKtoPXtD4J89wLEvXx6589-Vs&hl=en&sa=X&ved=0ahUKEwj2nvG42brPAhVI6mMKHYMnAakQ6AEISzAG#v=onepage&q=perfect%20anti% 20correlation & f = false spricht auch darüber.
@EmilioPisanty: eine andere - http://iopscience.iop.org/article/10.1088/1367-2630/14/5/053030/meta "Ungleichung (1) kann durch eine große Klasse von verschränkten Zuständen verletzt werden. Zum Beispiel, wenn Aliceund Bob teilen sich einen Singulettzustand Ψ− = ((| 10〉 - | 01〉) / √2), und Alice hat perfekte Detektoren, sie kann die maximale Verletzung von S = 3 erreichen, da identische Messungen am Singulettzustand immer führenzu perfekt antikorrelierten Ergebnissen "
@kpv Worum geht es dir?Sind Sie nicht in der Lage, den Unterschied zwischen den Behauptungen "Alle verschränkten Zustände repräsentieren antikorrelierte Spins" und "Einige verschränkte Zustände repräsentieren antikorrelierte Spins" zu erkennen?Warum fühlen Sie sich berechtigt, meine Zeit darauf zu verwenden, Ihnen die Verstrickung zu erklären, wenn Sie sich weigern, ein Lehrbuch in die Hand zu nehmen?
@EmilioPisanty: Es sieht so aus, als ob es einige Zustände gibt, die eine perfekte Antikorrelation aufweisen.Wenn ja, dann wurde mir klar, was die Verwirrung ist.Wenn wir 1 Million Paare in diesem Zustand vorbereiten, zeigen alle 1 Million Paare entgegengesetzten Spin, wenn sie im gleichen Winkel gemessen werden.Wiederum meine ich nicht in jedem Zustand, ich meine, es gibt einige Zustände."Wenn wir in diesem Zustand 1 Million Paare vorbereiten, zeigen alle 1 Million Paare entgegengesetzten Spin, wenn sie im gleichen Winkel gemessen werden."Das meine ich mit "immer antikorreliert".Einige Leute bekommen es sofort, andere diskutieren über Staaten.
@EmilioPisanty: Angesichts des Zustands der perfekten Antikorrelation ist die Antikorrelation nicht statistisch, sondern muss durch ein Gesetz und nicht durch eine Wahrscheinlichkeit durchgesetzt werden.
@EmilioPisanty: Ich habe keinen guten Grund zu antworten, warum ich nicht zu einem Lehrbuch komme.Aber ich werde versuchen, "zu erklären, überhaupt nicht zu rechtfertigen".Ich hätte ein Lehrbuch in die Hand genommen, wenn ich die Genauigkeit der Mathematik auch nur um ein Jota angezweifelt hätte.Ich bezweifle überhaupt nicht die Genauigkeit der Mathematik und ich gehe davon aus, dass alles perfekt ist, ohne es überhaupt anzusehen.Das Thema, das ich untersuche, ist die "Realität" dieser Mathematik."Alle Realität wird durch genaue Mathematik beschrieben, aber jede genaue Mathematik beschreibt nicht unbedingt die Realität."


Diese Fragen und Antworten wurden automatisch aus der englischen Sprache übersetzt.Der ursprüngliche Inhalt ist auf stackexchange verfügbar. Wir danken ihm für die cc by-sa 3.0-Lizenz, unter der er vertrieben wird.
Loading...